FNCE 3030 quiz questions

अब Quizwiz के साथ अपने होमवर्क और परीक्षाओं को एस करें!

The greater an investor's belief in market inefficiency, then the greater the argument for: -Investing in market indexes -Passive investing -Buy and hold -Active investing

Active Investing

What is the intrinsic value of a stock option given the following information? Option premium $1.50 Strike price $34 Current stock price $32 Time value = $1.50 -$0 -$1.50 -$2.00 -$3.50

$0

What is the bid-ask spread given the following market-maker quote: 45.18-45.24 [1x10]? -$0.01 -$0.06 -$0.10 -$0.11

$0.06

Brett opened a margin account with a discount brokerage firm. Three months ago, he sold short 100 shares of stock; the market price of the stock at that time was $63.50. Today it is priced at $47.30. If he decides to "buy to close" (i.e., buy 100 shares of stock in order to close his open "short position") what will be his net gain or loss? (For purposes of this problem assume that the cost of opening the position was $5, the cost of closing the position was $5 and margin interest was $40.) -$1,620 gain -$1,570 gain -$33.80 loss -$1,620 loss

$1,570 gain

Your client sold an oil painting for $52,600. The artwork was purchased 10 years ago for $10,000. The cost of the sale was 10%. Given that your client is in the 32% bracket for ordinary taxes, then what is her tax liability? -$11,949 -$14,728 -$16,832 -$10,455

$10,455

What is the price of a callable $100,000, 4.8% semi-annual bond that matures in 18 years and is callable in 10 years, given a current market yield on similar debt of 5.4 and a call premium of 10%? -$101,279.77 -$95,410.41 -$101,365.74 -$96,979.38

$101,279.77

Your client is interested in buying a real estate property. The net operating income from the property is projected to be $12,500 for the first year. That NOI is expected to grow at a rate of 2% per year after that. What is the maximum price that your client should pay for the property if their required rate of return is 9.2%? -$173,611 -$208,333 -$135,870 -$115,000

$173,611

The current dividend (Do) for Company A is $1.5400 per share. Dividends have been growing at a constant growth rate of 3% per year and that trend is expected to continue. If the required rate of return is 11.5%, then what is the maximum price than an investor should pay for the stock? -$13.39 -$18.66 -$10.62 -$18.11

$18.66

Dollar cost averaging is an investment strategy whereby you invest a fixed dollar amount in a security at predetermined intervals. The premise is that more shares are purchased when the market is down, which over the long-term creates an average cost per share lower than just the arithmetic average of prices. If your annual $5,000 contributions to the ABC Mutual Stock Fund occurred over five years, as depicted below, then what is your dollar cost average per share? Year 1 share price = $25 Year 2 share price = $28.50 Year 3 share price = 24.25 Year 4 share price = 19.75 Year 5 share price = 22.60 -$23.67 -$24.13 -$24.02 -$24.25

$23.67

An investor has a concentrated stock position in ABC stock of 10,000 shares. How much would it cost to hedge that position given that the ask price for the February ABC $50 put option is $2.50? -$5,000 -$12,500 -$25,000 -$50,000

$25,000

An investor purchases 1 ABC July 25 call option with a premium of $1.25. At what market price will the investor "break-even?" -$23.75 -$25.00 -$26.25 -$27.50

$26.25

What is the premium on five (5) March 105 call contracts given the following option price quotations? Stock Price: 100 Strike: 105 Expiration: March Ask: 8.85 Stock Price: 100 Strike: 110 Expiration: March Ask: 4.25 Stock Price: 100 Strike: 115 Expiration: March Ask: 1.65 -$8.85 -$44.25 -$885.00 -$4,425.00

$4,425.00

Given that an investor maintains $500,000 in a brokerage account, with $350,000 in cash and $150,000 in securities (held in street name), then what would be recovered in the event of SIPC-insured brokerage failure? -$250,000 -$350,000 -$400,000 -$500,000

$400,000

What is the price of a preferred stock given that it pays a cumulative preferred dividend of $3.25, if the market required rate of return is 7.4%? -$34.91 -$65.00 -$43.92 -not enough information is given

$43.92

An investor deposited $300,000 in different non-retirement accounts (checking account, savings account and certificates of deposit) that were titled as single owner. The investor also deposited $200,000 in an IRA account. How much would the bank customer recover in the event of a bank default? -$200,000 -$300,000 -$450,000 -$500,000

$450,000

What is the maximum profit that an investor could earn by buying one put option contract at a strike price of $60, given that the premium price is $5.50? -$5,450 -$6,000 -$6,550 -Unlimited profit potential in owning the option

$5,450

What is the maximum price that an investor should pay for a 10-year, $10,000 zero coupon bond, if the current yield to maturity on comparable debt is 4.375%? -$6,516.81 -$6,139.13 -$6,755.64 -$10,000.00

$6,516.81

What is the value of a company based on the following data and a discount rate of 8%? -$7,806,411 -$7,746,177 -$7,929,896 -$11,100,054

$7,746,177

What is the price of a $100,000, 4.8% semi-annual coupon bond that matures in 12 years, if the current yield to maturity is 5.4%? -$96,959.61 -$94,751.17 -$94,800.01 -$105,425.25

$94,751.17

In reviewing the portfolio of a new client, you notice that she has a meaningful investment in Treasury Inflation-Adjusted Securities (TIPS). Each of the following statements is correct, except for one. Which statement is incorrect? -TIPS make semi-annual coupon payments -TIPS pay a fixed rate of interest but the interest is paid on the adjusted principal -The principal (i.e., face value of the bond) is adjusted for the rise or fall in the Consumer Price Index -The investor is only taxed on the interest payment; the principal adjustment is taxed once the security is sold or matures

-The investor is only taxed on the interest payment; the principal adjustment is taxed once the security is sold or matures

Based on the following data, what is the Stock A's Sharpe Ratio? Portfolio return = 12.6% Standard deviation = 26.3% Beta = 1.4 Risk-free rate = 3.2% -.60 -.67 -.11 -.36

.36

What is the correlation coefficient, if Security M has a standard deviation of 21.8%, Security P has a standard deviation of 14.6% and the covariance is 2.1%? -.45 -.31 -.89 -.66

.66

The correlation coefficient describes how the two assets move in price relative to one another. What is the covariance of Stock A and B, given the following information? Stock A - 12.5% expected return, 18.6% standard deviation Stock B - 8.6% expected return, 13.2% standard deviation Correlation coefficient = .7 -0.007525 -171.864 -0.07112 -0.017186

0.017186

Luke is the portfolio manager for the Front Range Value Fund. Last year his portfolio return was 9%, and the portfolio had a beta of 1.2. The S&P 500 Index had a 7.5% return. The T-bill rate was 2.3%. What is the alpha of the Fund? -2.76 -0.46 -0.00 -1.50

0.46

The T-Bill yields 2.5%. The optimal risky portfolio has a return of 12% and the standard of deviation of 16%. What is the slope of the capital market line (i.e., reward-to-volatility ratio)? -0.7500 -1.3333 -0.5938 -1.1250

0.5938

What is the beta of Security A given the following data? Standard deviation of market = .165 Variance of Security A = .0328 Correlation coefficient = .85 -0.77 -0.65 -0.17 -0.93

0.93

Which of the following strategies might immunize a portfolio for an investment horizon of 14 years? (1) Purchase several coupon paying bonds with maturities ranging from 16-18 years. (2) Purchase a zero-coupon bond maturing in 20 years. (3) Create a mixed portfolio of debt maturing in 14 years and equity securities. (4) Purchase a zero-coupon bond maturing in 14 years. -2 and 3 -1 and 4 -4 only -1 and 3

1 and 4

Which of the following reasons support an investor's decision to add alternative investments to the overall portfolio? (1) Better risk-return profile (2) Improve diversification (3) Introduce leverage into the portfolio (4) Reduce portfolio costs (5) Improve the portfolio's liquidity -1, 2 and 3 -1 and 3 -2 and 5 -1, 4 and 5

1, 2 and 3

Which of the following should occur as the business cycle moves from peak to recession? (1) Unemployment rises as corporations have to lay-off workers. (2) Fiscal and monetary policy shifts from controlling inflation to providing economic stimulus. (3) Economic indicators will turn negative; however, leading indicators will turn positive later in the recession as the economy reaches the trough of the cycle. (4) Stocks and bonds will both decrease in value. -1 and 2 -3 and 4 -1, 2 and 3 -1, 2, 3 and 4

1, 2 and 3

Which of the following are a bullish sign for technical analysis? (1) The stock remains above the long-term trend line (2) The majority of analysts on the stock are bearish. (3) The stock is approaching an overhead line of resistance (4) The stock continues to find support on the 200-day moving average line. (5) The stock is rising on low volume and dropping on high volume. -1 and 3 -1, 3 and 5 -1, 2 and 4 -2 and 4

1, 2 and 4

Which of the following risks are associated with investors holding callable bonds? (1) interest rate risk (2) reinvestment risk (3) purchasing power risk (4) default risk (5) call risk (6) liquidity risk -2, 5 and 6 -1, 3 and 4 -1, 2, 3, 4, 5 and 6 -1 and 5

1, 2, 3, 4, 5 and 6

Which of the following individual client factors influence the appropriate asset allocation mix? (1) Risk tolerance (2) Financial need (3) Time horizon (4) Financial capacity (5) Investment knowledge and experience (6) Individual preferences and constraints -3, 4 and 5 -1, 2 and 6 -1, 2, 3, 4, 5 and 6 -1, 2, 3 and 6

1, 2, 3, 4, 5, and 6

Janice owns 1000 shares of Microsoft stock in a margin account. She writes 10 covered call options (American options) at a strike price of $30. Elissa purchases the 10 call options at a premium price of $1.45. Which of the following statements are true regarding this transaction? (1) Janice, the call writer, receives the premium payment from Elissa (2) Elissa, the call buyer, is obligated to exercise the call option sometime before expiration (3) Janice is obligated to perform (i.e., deliver the shares) if Elissa elects to exercise the 10 call options anytime up to expiration (4) Elissa has the right to call away the shares from Janice or to sell the call options to someone else up to the time of expiration -1 and 2 -3 and 4 -1, 3 and 4 -1, 2, 3 and 4

1, 3 and 4

Which of the following statements regarding private placement debt are correct? (1) Private placement securities are sold to accredited investors (2) Standard debt terms and schedules (3) Higher interest than earned on similar public security (4) High liquidity (5) Offered by mid- to larger-capitalized companies -2, 4 and 5 -3 and 4 -1 and 2 -1, 3 and 5

1, 3 and 5

What is the portfolio's beta given the following Security A, $300,000 position, beta = .86 Security B, $550,000 position, beta = 1.12 Security C, $150,000 position, beta = 1.42 -1.09 -1.27 -0.98 -0.45 -1.13

1.09

You are deciding between two Securities, X and Y; however, you want to add the security that offers the best "risk-return" profile. Based on the following data, calculate the coefficient of variation and identify the best security selection. Security A - expected return of 10.2% and standard deviation of 16.4% Security B - expected return of 8.3 and standard deviation of 11.9 -1.6 and 1.4, respectively. Select Security B because it has a lower coefficient of variance. -0.62 and 0.70, respectively. Select Security B because it has a higher coefficient of variance. -0.62 and 0.70, respectively. Select Security A because it has a lower coefficient of variance. -1.6 and 1.4, respectively. Select Security A because it has a higher coefficient of variance.

1.6 and 1.4, respectively. Select Security B because it has a lower coefficient of variance.

Given the following information, what is the expected return for the 2-stock portfolio? Stock A - 12.5% expected return, 18.6% standard deviation Stock B - 8.6% expected return, 13.2% standard deviation Dollar value of Stock A = $48,000 Dollar value of Stock B = $72,000 -10.16% -10.94% -12.60% -15.36%

10.16%

Given the following information, what is the standard deviation for Security A? 30% probability of a bull market, which will offer a 24% return 50% probability of a neutral market, which will produce an 8% return 20% probability of a bear market, which will offer a -9% return -7.2% -1.3% -6.3% -11.5%

11.5%

Stock B is selling for $29.50. The company's next annual dividend is projected to be $1.95 per share. Dividend growth has been at a steady rate of 5%, which is expected to continue. If you buy Stock B at the current price, then what is the rate of return? -11.61% -11.94% -12.21% -15.21%

11.61%

Your client has a three-year investment horizon; therefore, you decide to use the last three years of historical market data in determining your expected return assumption. Based on the holding period returns, what is the geometric (time weighted) annual rate of return assumption you will use when entering data into your portfolio optimization analyzer? (Base your response on the following market data. Also, calculate your answer based on four decimal places and round your final answer to two decimal places.) 3 years ago (begin price = $23.50; end price = $28.00) 2 years ago (begin price = $28.00; end price = $26.25) 1 year ago (begin price = $26.25; end price = $33.75) -12.81% -17.61% -14.54% -9.80%

12.8%

Your client has a three-year investment horizon; therefore, you decide to use the last three years of historical market data in determining your expected return assumption. Based on the holding period returns, what is the arithmetic (time weighted) annual rate of return assumption you will use when entering data into your portfolio optimization analyzer? (Base your response on the following market data.) 3 years ago (begin price = $23.50; end price = $28.00) 2 years ago (begin price = $28.00; end price = $26.25) 1 year ago (begin price = $26.25; end price = $33.75) -9.8% -10.5% -12.8% -13.8%

13.8%

When holding a risky security, you are entitled to a risk premium. Given the following information, what is your expected risk premium for holding a risky asset, based on the security market line formula? Market return = 12% Risk-free rate = 2% Standard deviation = 22% Beta = 1.4 -14.6% -10% -12% -14.0% -12.2% -16.8%

14.0%

Based on the capital asset pricing model, what is the required rate of return on a security given the following information? Security's beta is 1.6 risk-free rate of interest is 2.8% market portfolio has an expected return of 11.2% -20.7% -13.4% -16.2% -10.6%

16.2%

Jake invested $10,000 in a mutual fund that returned 12% the first year. The second year he invested another $8,000 and the fund was up another 15%. Pleased with the growth of investments, he invested another $9,000 in year three and enjoyed another 9% increase. At the end of that year, the portfolio was valued at $37,460. What was Jake's average dollar-weighted return? -26% -12% -49% -17%

17%

Which of the following assets would help lower the overall portfolio risk based on the correlation coefficient? Security with a correlation coefficient of +1 to the portfolio Security with a correlation coefficient of +.7 to the portfolio Security with a correlation coefficient of 0 to the portfolio Security with a correlation coefficient of -.3 to the portfolio -3 and 4 -2, 3 and 4 -4 only -3 only -1, 2, 3 and 4

2, 3 and 4

Your client asks you to explain warrants because he is considering the purchase of a corporate bond with warrants. Which of the following explanations would you be correct in telling him? (1) With a warrant, he would be able to exchange the bond for common shares in the company. (2) Many warrants are detachable, meaning they could be sold separately. (3) Warrants are issued by the company as a way to cheaply raise future equity capital (4) A warrant typically has an expiration within one or two years. (5) The price at which the bond with warrant could be bought would generally be more than purchasing just the straight bond (without warrant). -2, 3 and 5 -1, 3 and 4 -1 and 2 -3, 4 and 5

2, 3 and 5

What is the five-year spot rate, given a short rate of 2.24% in year one, a short rate of 2.48% in year two, a short rate of 2.62% in year three, a short rate of 3.05% in year four and a short rate of 3.62% in year five? -2.62% -2.80% -2.94% -3.05%

2.8%

Your client is considering a corporate bond with a 6.17% before-tax yield and a municipal bond with a 4.93% before-tax yield. At what marginal tax rate would the investor be indifferent between investing in the corporate and investing in the municipal bond? -28% -25.2% -20.1% -15.4%

20.1%

The S&P500 Index stands at 4,682. Given that the S&P500 contract has a multiplier of 250 and the mini-S&P500 contract has a multiplier of 50, how many e-mini S&P500 contracts are needed to hedge a portfolio valued at $5,000,000? (Round to the nearest contract.) -4 e-mini S&P500 contracts -5 e-mini S&P500 contracts -21 e-mini S&P500 contracts -213 e-mini S&P500 contracts

21 e-mini S&P500 contracts

Based on the following data, what is the expected return of Stock A? Treynor ratio = .156 Standard deviation = .22 Beta = 1.2 Risk-free rate = 6% -12.7% -18.8% -9.4% -24.7%

24.7%

Given the following information, what is the standard deviation for the 2-stock portfolio? Stock H - 22% standard deviation, portfolio weighting 36.4% Stock I - 32% standard deviation, portfolio weighting 63.6% Correlation coefficient - 0.7 Covariance - .049 -26.6% -21.9% -22.6% -32.5%

26.6%

The coefficient of determination explains how much of the movement of one security is explained by the movement of another. Which of the following statements are true given that the correlation coefficient between Security H and the S&P500 is .60? (1) 60% of the variation in return for Security H is explained by the movement of the S&P500 Index (2) 40% of the variation in return for Security H is explained by other factors (3) 36% of the variation in return for Security H is explained by the movement of the S&P500 Index (4) 64% of the variation in return for Security H is explained by other factors -1 and 2 -3 only -1 only -3 and 4

3 and 4

Given the following Treasury security data, then what is the best estimate of the one-year implied forward rate three years from now? 2.91% 3.12% 3.20% 3.30%

3.3%

Over the past year you earned a nominal rate of interest of 6.4 percent on your money. The inflation rate was 2.8 percent over the same period. What is the exact real rate of return that was earned? -3.40% -3.60% -3.50% -3.70%

3.5%

Given that the 3-year spot rate is 3.68%, the short rate in year one is 3.48% and the short rate in year two is 3.62%, then what is the forward rate for year 3? -3.59% -3.76% -3.94% -4.12%

3.94%

What is the duration of a $10,000 6% bond that matures in 5 years if the market yield to maturity is 6.4%? -4.49 years -4.32 years -4.46 years -4.53 years

4.46

What is the investor's 3-year holding period return given the following information? Initial investment $20,000 Year 1 cash distributions $1,200 Year 2 cash distributions $1,500 Year 3 cash distributions $800 End-of-year-3 sale proceeds $24,600 -17.5% -14.2% -40.5% -32.9%

40.5%

What is the yield to maturity of a $10,000, 6.2% semi-annual bond that matures in 15 years, given a current market price of $9,531.43? -6.70% -6.20% -3.35% -5.99%

6.70%

Your clients are fairly aggressive, when it comes to risk tolerance. Their time horizon is long-term. Which of the following portfolio allocations would be most appropriate based on CFP Board curriculum? -55% equity, 20% fixed income and 25% alternative investments -60% equity, 30% fixed income and 10% alternative investments -45% equity, 15 fixed income and 30% alternative investments -20% equity, 40% fixed income and 40% alternative investments

60% equity, 30% fixed income and 10% alternative investments

Given the following information, what is the expected return for Security A? 30% probability of a bull market, which will offer a 24% return 50% probability of a neutral market, which will produce an 8% return 20% probability of a bear market, which will offer a -9% return -9.4% -6.1% -7.7% -13.0%

9.4%

Which of the following correctly describes a bond ladder? -A bond that has a variable maturity date. -A fixed income portfolio where the individual bonds are purchased at different times but mature on the same date. -A fixed income portfolio where a portion of the portfolio is invested in short-term bonds and the remaining portion in long-term bonds. -A fixed income portfolio where the individual bonds mature on different dates.

A fixed income portfolio where the individual bonds mature on different dates.

Which of the following statements regarding municipal bonds is INCORRECT? -A municipal revenue bond is backed by the taxing power of the state or local government. -The yield to maturity on a municipal bond would be lower than the yield to maturity on a corporate bond of similar duration and credit quality. -Interest from municipal bonds is tax exempt for purposes of regular Federal income taxes (Form 1040). -Interest from municipal bonds issued by the resident's state would be tax exempt for state income tax purposes.

A municipal revenue bond is backed by the taxing power of the state or local government.

Which of the following statements is correct? -A stock right allows existing common shareholders to buy additional shares at a set price. -A callable bond allows the issuing company to recall the bond at any time before maturity. -A convertible bond allows the investor to retain the bond and buy a set number of shares at a price specified in the bond's provisions. -A bond with warrant allows the investor to turn in the bond to the issuing company in exchange for a set number of company shares as specified in the bond's provisions.

A stock right allows existing common shareholders to buy additional shares at a set price.

According to the Vanguard study entitled "Putting a value on your value," which of the following represents the greatest component of alpha (i.e., value to client)? -Identifying a suitable asset allocation -Formulating a workable budget -Behavioral coaching -Rebalancing the portfolio mix -Cost effective implementation

Behavioral coaching

Which of the following option strategies would be appropriate given that the investor wants to have the right to purchase a set number of shares at a pre-determined price? -Buy a put option -Write (sell) a call option -Write (sell) a put option -Buy a call option

Buy a call option

The SEC regulates all of the following, except for one. Which does NOT fall under the jurisdiction of the Securities and Exchange Commission? -Equities -Bonds -Commodities -Options

Commodities

If the company wants to offer shareholders ownership in company growth and voting rights, which of the following securities would it issue? Participating preferred shares Preferred shares Common shares Callable debt

Common Shares

All but one of the following are included within the index of leading economic indicators. Which of the following indicators is NOT a leading indicator of economic activity? -Average weekly claims for unemployment insurance -New manufacturing orders for consumer goods -Consumer price index -Number of new building permits -S&P 500 stock prices

Consumer price index

Your client's objective is to invest in a security that would provide the potential for growth and income. Which of the following assets would best accomplish this goal? -Income REIT (i.e., holds portfolios of mortgages and loans for construction) -Investment-grade corporate bond ETF -GIC -Non-participating preferred stock -Convertible bond fund

Convertible Bond Fund

A market participant who is responsible to holding and safeguarding investor assets is a -transfer agent -clearinghouse -custodian -underwriter

Custodian

Which of the following combinations of monetary and fiscal policy would promote an economic expansion? -Increase bank reserve requirements; decrease government spending -Decrease the discount rate; decrease individual income tax rates -Buy government securities (open market operations); increase corporate tax rates -Increase the discount rate; increase individual tax rates

Decrease the discount rate; decrease individual income tax rates

Which of the following statements about yield curves is INCORRECT? -A yield curve depicts that market yield to maturity for similar bonds across time. -During a recession you would expect the spread between investment-grade bonds and below-investment-grade bonds to be compressed (narrower). -You would expect to see the flattening of a normal yield curve given market expectations of an impending economic recession. -The yield curve for a U.S. Treasury bond would look different from that of a BBB-rated corporate bond.

During a recession you would expect the spread between investment-grade bonds and below-investment-grade bonds to be compressed (narrower).

According to the mean-variance criterion, which one of the following portfolios dominates all others (i.e., highest reward-to-risk ratio)? -E(r) = 15% with variance = 25% -E(r) = 18% with variance = 28% -E(r) = 20% with variance = 32% -E(r) = 12% with variance = 22%

E(r) = 18% with variance = 28%

Which of the following sectors would you expect to outperform during a market recession? -Auto manufacturer -Financial firm -Airline -Hospitality company -Food and beverage firm

Food and beverage firm

Which of the following hedge funds offer the best return-to-risk profile based on William Sharpe's information ratio given the fact that the benchmark offered an 8.5% return? (1) Fund A - 17.0% return, tracking error 6.8%, beta 1.8 (2) Fund B - 24.0% return, tracking error 9.3%, beta 2.1 (3) Fund C - 28.0% return, tracking error 12.8%, beta 2.7 (4) Fund D - 10.0% return, tracking error 3.5%, beta 1.3 -Fund A -Fund C -Fund B -Fund D

Fund B

Which of the following cognitive biases help explain why individuals value current consumption more than deferred gratification? -Ambiguity bias -Status quo or endowment effect -Anchoring bias -Over-conservatism bias -Hyperbolic discounting

Hyperbolic discounting

Your client seeks exposure to the real estate market. Given that the client's objectives are high monthly income and capital preservation, which of the following would be most appropriate? -Income REIT -Equity REIT -Hybrid REIT -Foreign REIT

Income REIT

An investor buys 2,000 shares a stock at $20 on margin. If the margin rate = 50% and the maintenance margin is 35%, how much are the initial margin required and the maintenance call price? -Initial margin = $14,000; maintenance -call price = $15.38 -Initial margin = $14,000; maintenance call price = $14.29 -Initial margin = $20,000; maintenance call price = $14.29 -Initial margin = $20,000; maintenance call price = $15.38

Initial margin = $20,000; maintenance call price = $15.38

In a highly efficient market, there should be no exceptions. However, evidence shows some persistent patterns that offer out-performance relative to a buy and hold strategy. Which of the following is NOT recognized market anomalies? -Turn-of-the-month effect -January effect -"Dogs of the Dow" -International stock effect -Small firm effect -Low market multiple stocks

International stock effect

Jeff applies a core and satellite investment approach, when his investing retirement portfolio. Which of the following would be representative of this strategy? -Invest 100% of the investment portfolio in a broad market index. -Invest 50% of the investment portfolio in an actively managed equity fund and 50% in an actively managed bond fund. -Invest 60% of the investment portfolio in a combination of mutual index funds, 20% in an actively managed emerging markets fund and 20% in individual small- and mid-cap stocks, which are actively traded. -Invest 70% of the investment portfolio in a S&P500 index ETF and 30% in the MSCI EAFE.

Invest 60% of the investment portfolio in a combination of mutual index funds, 20% in an actively managed emerging markets fund and 20% in individual small- and mid-cap stocks, which are actively traded.

Which of the following market participants is NOT regulated by FINRA? -Broker-dealers -Registered securities representatives -Investment advisers -Crowd-funding intermediaries

Investment advisers

Which of the following market participants would NOT be considered an accredited investor? -Registered investment adviser -Investor with more an $1M in assets (not including the personal residence) -Investor with income of $100,000 over the last two years and income in the current year expect to exceed the $100,000 -Bank

Investor with income of $100,000 over the last two years and income in the current year expect to exceed the $100,000

Which form of efficient market hypothesis is supported by the January effect? -Weak form efficiency -Strong form efficiency -Semi-strong form efficiency -None of these

None of these

Your friend completed the Market Technician Association certification in technical analysis. Which form of market efficiency supports his efforts in applying the tenants of technical analysis to outperform? -Strong form efficiency -None of these -Weak form efficiency -Semi-strong form efficiency

None of these

Which of the following statements correctly describe the difference between open- and closed-end mutual funds? -Open-end funds offer a set number of shares, while a closed-end fund offers an unlimited number of shares -An open-end fund will never have a load, while closed-end funds may or may not charge a sales load. -An open-end fund can be sold short, which closed-end funds cannot be sold short. -Open-end mutual funds are bought and sold at their net asset value, while a closed-end fund trade on stock exchanges based on supply and demand

Open-end mutual funds are bought and sold at their net asset value, while a closed-end fund trade on stock exchanges based on supply and demand

Which of the following statements is correct regarding asset allocation and diversification? -Proper asset allocation and diversification within each asset category can eliminate idiosyncratic (unsystematic risk). -Standard deviation is the correct measure to use when assessing a multi-asset portfolio risk. -Adding two securities that have a correlation coefficient of one will lower portfolio risk. -Diversification will lower both expected risk and expected return.

Proper asset allocation and diversification within each asset category can eliminate idiosyncratic (unsystematic risk).

Your client is interested in adding alternative investments to the portfolio. Which of the following would offer the lowest level of liquidity? -REIT -Currency ETF -RELP -MLP

RELP

Market volatility creates an on-going roller-coaster of emotion. Which of the following emotional or cognitive biases best describe the sabotaging behavior of hanging onto losers, rather than harvesting the tax loss and then re-positioning the portfolio? -Mental accounting -Herding -Ambiguity aversion -Representativeness -Regret avoidance

Regret avoidance

Which of the following investors does the SEC seek to protect, when it comes to financial securities? -Retail investors -Institutional investors -Accredited investors -Qualified institutional buyer

Retail investors

Which of the following is NOT a fundamental premise of prospect theory. -When experiencing a loss, risk-averse investors will become more risk seeking -Investors focus on change in wealth rather than wealth maximization -Risk-averse investors are more focused on gains than losses -The highest portfolio value becomes a reference point for portfolio evaluation

Risk-averse investors are more focused on gains than losses

You manage a $50,000,000 stock portfolio for your institutiional client. The client is very concerned about a market sell-off and wants to hedge the long portfolio. Which of the following futures indexes would best be used as the hedging instrument? (Remember - r-squared is the correlation of determination) -S&P100 Index, r-squared .54 -S&P500 Index, r-squared .92 -Russell 3000 Index, r-squared .84 -DJI index, r-squared .45

S&P500 Index, r-squared .92

What is the name of the securities market where individual investors buy and sell securities on a physical or virtual stock exchange? -primary market -secondary market -third market -fourth market

Secondary market

An investor recently profited from a series of trades. A new opportunity is considered, and the investor quickly decides on investing a large percentage of the portfolio. What heuristic probably impacted this aggressive investment decision. -Framing -Mental accounting -Sequential prospects -Endowment effect

Sequential prospects

A stock trades on the New York Stock Exchange. When is the new buyer of the stock considered the owner? -Trade day -Ex-dividend day -Settlement day -End of month

Settlement day

Risk versus return is a core concept of investments. Based on historical evidence, which of the following asset categories would offer the highest expected long-term return of return with the highest expected volatility? Small-cap stocks Real estate Large-cap stocks High-yield corporate bonds U.S. Treasury bills

Small Cap Stock

Which of the following statements is correct regarding systematic and unsystematic risk? -Inflation is an example of unsystematic risk -Business and financial risk are both major systematic risks. -Systematic risk can be reduced by owning a diversified portfolio of securities -Standard deviation is a measure of both unsystematic and systematic risk

Standard deviation is a measure of both unsystematic and systematic risk

Holding all other factors constant, which of the following securities would be most attractive to a portfolio manager who looks for value? -Stock K = 2.25M shares, annual earnings of $2,772,000, current price $26.50 -Stock L = 5M shares, annual earnings of $2,580,000, current price $11.50 -Stock M = 10M shares, annual earnings of $5,650,000, current price $14.10 -Stock N = 3M shares, annual earnings of $2,465,000, current price $15.40 -Stock N -Stock M -Stock K -Stock L

Stock N

What security would a company issue, given that its purpose is to raise additional equity capital within a particular period of time without having to go through the underwriting costs of floating a new round of financing? -Preferred stock offering -Stock rights -Debt issue -Repurchase agreement

Stock Rights

All of the following statements regarding the capital market line (CML) are correct except for one. Which statement is incorrect? -The CML is drawn on a graph that depicts the expected market return on the vertical axis and the standard deviation of the market on the horizontal axis. -The slope of the CML is defined as the market return premium (expected return of the market less the risk free rate) divided by the standard deviation of the market. -The CML is drawn from the risk-free rate to the optimal risky market portfolio (point of tangency on the efficient frontier). -The CML describes the relationship between the market's expected return and systematic risk.

The CML describes the relationship between the market's expected return and systematic risk.

Which of the following statements best describes the arbitrage pricing model? -The arbitrage pricing model is a multi-factor pricing model that considers the security's sensitivity to various macroeconomic variables -The arbitrage pricing model is graphically depicted by the slope of the capital market line -The arbitrage pricing model is graphically depicted by the slope of the security market line -The arbitrage pricing model is a single factor model based on a security's risk as measured by beta.

The arbitrage pricing model is a multi-factor pricing model that considers the security's sensitivity to various macroeconomic variables

All but one of the following statements are correct regarding dividend reinvestment plans (DRIPs). Which of the following statements is incorrect? -The dividends are not reported as taxable income because the money is not received. -A company might consider adopting a DRIP because it is a way to raise additional equity capital at little to no expense. -In most cases, no commissions are charged with the DRIP. -When an investor signs up for a DRIP, dividends are automatically used to purchase additional shares in the company.

The dividends are not reported as taxable income because the money is not received.

Which of the following statements is true regarding the indifference curve of a risk-averse investor? -The indifference curve for a risk-averse investor is flatter than for a risk-neutral or risk-loving investor -The investor's indifference curve depicts the same expected returns and different standard deviations -The investor's indifference curve depicts all combinations of risk and return offering the same level of utility -The investor's indifference curve depicts the same standard deviations but different expected returns

The investor's indifference curve depicts all combinations of risk and return offering the same level of utility

Which of the following statements is INCORRECT? -The higher the initial yield to maturity of the bond, then the less sensitive the bond is to interest rate moves. -The price of existing bonds moves inversely with market interest rates. -The lower the coupon rate of the bond, then the lower the price sensitivity to interest rate moves. -The greater the term to maturity of the bond, then the greater the price sensitivity to interest rate moves.

The lower the coupon rate of the bond, then the lower the price sensitivity to interest rate moves.

Which of the following statements about unit investment trusts (UITs) is incorrect? -UITs typically consists of a portfolio of fixed-income securities. -The pooled investment allows individuals to participate in a diversified portfolio of debt securities. -The portfolio is actively managed, although the intent is for low to moderate turnover -Shares of the UIT can typically be redeemed with the investment company, as specified in the prospectus -The trusts are intended to self-liquidate.

The portfolio is actively managed, although the intent is for low to moderate turnover

Which of the following statements is INCORRECT, regarding exchange traded funds (ETFs)? -You can buy and sell ETFs throughout the day (just like a stock) -There is typically a front-end sales load for purchasing an ETF -Fees with ETFs are lower than most mutual funds -An ETF offers diversification since it is essentially a basket of securities

There is typically a front-end sales load for purchasing an ETF

In contrast to a forward contract, future contracts -Trade on organized exchanges -Have contract sizes that are tailored for investor needs -Have negotiable expiration dates -Must settle with delivery of the asset

Trade on organized exchanges

Which of the following securities sell at a discount to face value, rather than offering periodic coupon payments? -Corporate bond -U.S. Treasury Bill -TIPS -Eurodollar bond

U.S. Treasury Bill

Which of the following is classified as a bond (capital market security), rather than a cash equivalent security (money market security)? Certificate of deposit Commercial paper U.S. Treasury Bill U.S. Treasury Note

U.S. Treasury Note

Which of the following securities would have the lowest level of default risk? Preferred stock Corporate bond rated A1 by Moody's rating agency Common shares U.S. Treasury Note A-rated municipal bond

U.S. Treasury Note

When does a stock trade settle? -the same business day -after one business day (T+1) -after two business days (T+2) -after five business days (T+5)

after two business days (T+2)

According to the Brinson, Singer and Beebower study, which of the following is the most important decision that impacts long-term wealth accumulation? -asset allocation -portfolio hedging -security selection -market timing

asset allocation

Given the duration of a bond is 8.62 years, the current yield to maturity is 6.5%, and the yield to maturity is expected to decrease by 25 basis points (i.e., 0.25%) what is the expected percent price change of the bond? a) -3.32% b) +2.02% c) +3.32% d) -2.02%

b) +2.02%

What is the duration of a portfolio that contains the following securities? Bond F - $100,000, duration = 6.4 years Bond G - $250,000, duration = 11.8 years Bond H - $150,000, duration = 8.4 year a) 4.49 years b) 9.70 years c) 8.97 years d) 8.40 years e) 4.85 years

b) 9.70 years

An investor shorted 500 shares of ABC. Shortly after the trade, ABC dropped in price by $15. If the investor wanted to realize the gain by cashing in the short position, then what type of order would be placed? -sell order -buy order -sell to cover or sell to close order -buy to cover or buy to close order

buy to cover or buy to close order

Holding all other factors constant, what will an increase in implied volatility do to option prices? -no effect on pricing for markets already trading -cause the option price to decline -cause the option price to increase -no direct impact because historical volatility is the relevant variable for pricing

cause the option price to increase

Given the duration of a $1,000 bond is 8.75 years, the current yield to maturity is 6.5%, and that yield to maturity is expected to increase by 50 basis points (i.e., +.5%) what is the dollar change in price if the current price of the bond is $974.60? a) + $39.85 b) - $39.85 c) + $40.04 d) - $40.04

d) - $40.04

You find that the historical return and volatility of domestic large-cap growth stocks are 10.7% and 12.1%, respectively. What range of annual returns do you expect to realize 95% of the time? a) -1.4% to 22.8% b) -12.8 to 33.2% c) -25.6% to 47.0% d) -13.5% to 34.9%

d) -13.5% to 34.9%

According to bankruptcy code, what is the correct claim order in the event of a company liquidation (i.e., first priority, second priority, residual interest)? -common shareholders, preferred shareholders and debtholders -debtholders, preferred shareholders and common shareholders -preferred shareholders, debtholders and common shareholders -debtholders, common shareholders and preferred shareholders

debtholders, preferred shareholders and common shareholders

Based on the life cycle investment strategy, at which stage of the economic life cycle would the investor be weighted in fixed income securities? -never, the investment portfolio would always be weighted towards growth securities -accumulation stage (age 20-45) -consolidation stage (age 45-67) -distribution stage (in retirement)

distribution stage (in retirement)

A firm in an industry that is very sensitive to the business cycle will likely have a stock beta ___________. -greater than 1.0 -equal to 1.0 -less than 1.0 but greater than 0.0 -equal to or less than 0.0 -there is no relationship between beta and sensitivity to the business cycle

greater than 1.0

Which of the following statements is INCORRECT regarding hedge funds? -hedge funds are marketed to accredited investors, not the general public -withdrawals are restricted, including a potential lock-up period and semi-annual withdrawal windows -gains and losses flow through to investors -hedge fund information is easily accessible in online databases -costs typically include advisory fees and performance incentives

hedge fund information is easily accessible in online databases

A protective put strategy is -long a put option plus a long position in the underlying asset -long a put option plus long a call option on the same underlying asset -long a call option plus a short put on the same underlying asset -long a put option plus a short call on the same underlying asset

long a put option plus a long position in the underlying asset

What would you expect to see at the end of a recession? -high inflation and high unemployment -low interest rates and high unemployment -low inflation and low unemployment -high inflation and high unemployment

low interest rates and high unemployment

Which of the following is NOT a characteristic of managed future accounts? -varying correlation with equity funds -advisory fees plus performance incentives -reliance on technical analysis trading strategy -long and short positions -low volatility of return

low volatility of return

Which of the following mutual fund categories include pooled investments that attempt to specifically eliminate systematic risk by holding a carefully balanced position of longs and shorts within the same industry or sector? -long-short fund -bear market fund -market neutral fund -multi-alternative fund

market neutral fund

Due diligence is always an important element of investment. Which of the following securities would require the lowest level of due diligence before the investment? -market neutral mutual fund -individual hedge fund -private equity -venture capital firm

market neutral mutual fund

Which one of the following portfolios cannot lie on the efficient frontier as described by Markowitz? Portfolio A = E(r) 10%, standard deviation 21% Portfolio B = E(r) 6%, standard deviation 9% Portfolio C = E(r) 15%, standard deviation 32% Portfolio D = E(r) 12%, standard deviation 18% -only portfolio B cannot lie on the efficient frontier. -only portfolio D cannot lie on the efficient frontier. -only portfolio A cannot lie on the efficient frontier. -only portfolio C cannot lie on the efficient frontier. -not enough information is given

only portfolio A cannot lie on the efficient frontier.

If a corporate insider wants to sell a large block of public shares, then what type of offering this represent? -initial public offering -secondary offering -follow-on offering -rights offering

secondary offering

"Stock prices adjust rapidly to the release of all new public information." This statement is an expression of which one of the following ideas?" -random walk hypothesis -technical analysis -arbitrage pricing theory -semi-strong form of efficient market theory

semi-strong form of efficient market theory

The MSRB regulates all but one of the following market participants. Which one is not managed by the Municipal Securities Rulemaking Board? -municipal advisers -broker-dealers who trade or issue municipal securities -investment banking firms that are involved in municipal security issuance -state and local governments who are issuing the debt

state and local governments who are issuing the debt

Your friend, Ben, is an active investor who practices sector rotation based on the chart patterns of different sector ETFs. Right now he is over-weighted in consumer non-durable goods. This trading approach is an example of which of the following? -tactical asset allocation -strategic asset allocation -portfolio rebalancing -trading optimization

tactical asset allocation

The efficient frontier for various combination of risky assets describes: -the highest level of standard deviation for a given level of risk -the highest level of return for a given level of risk -the lowest level of beta for a given level of return -the most cost-effective portfolio

the highest level of return for a given level of risk

A covered call strategy is -the simultaneous purchase of the call and the underlying asset. -the purchase of a share of stock with a simultaneous sale of a put on that stock. -the short sale of a share of stock with a simultaneous sale of a call on that stock. -the purchase of a share of stock with a simultaneous sale of a call on that stock.

the purchase of a share of stock with a simultaneous sale of a call on that stock.

Investor A sells 1000 shares to Investor B. What market participant is responsible for issuing and cancelling certificates of share ownership? -transfer agent -clearinghouse -custodian -depository

transfer agent

Which of the following ratios describe the estimated price change of the option premium due to a change in market expectations of volatility? -delta -gamma -theta -vega -rho

vega


संबंधित स्टडी सेट्स

Chapter Fourteen Review + Quiz Questions

View Set

Investments Chp 3,4 market microstructure and investment companies

View Set

Ch. 14: CPT: twist drill, burr holes or trephine, craniectomy, or craniotomy

View Set

CH. 3 Cell Structure and Function

View Set

HOMONYMS, HOMOPHONES, HOMOGRAPHS and SYNONYMS, Homophones Homographs Homonyms

View Set

Lending Info and Do Not Call Laws

View Set